Difference between revisions of "1965 AHSME Problems/Problem 3"

(Created page with "== Problem == The expression <math>(81)^{-2^{-2}}</math> has the same value as: <math>\textbf{(A)}\ \frac {1}{81} \qquad \textbf{() }\ \frac {1}{3} \qquad \textbf{(C) }\ 3 \...")
 
m (Added (B) choice)
Line 3: Line 3:
 
The expression <math>(81)^{-2^{-2}}</math> has the same value as:
 
The expression <math>(81)^{-2^{-2}}</math> has the same value as:
  
<math>\textbf{(A)}\ \frac {1}{81} \qquad \textbf{() }\ \frac {1}{3} \qquad \textbf{(C) }\ 3 \qquad \textbf{(D) }\ 81\qquad \textbf{(E) }\ 81^4</math>
+
<math>\textbf{(A)}\ \frac {1}{81} \qquad \textbf{(B) }\ \frac {1}{3} \qquad \textbf{(C) }\ 3 \qquad \textbf{(D) }\ 81\qquad \textbf{(E) }\ 81^4</math>
  
 
== Solution ==
 
== Solution ==
  
 
We know that <math>81^{-2^{-2}}</math> is equivalent to <math>81^{\frac{1}{-2^2}}=81^{\frac{1}{4}}</math>, which is the same as <math>\sqrt[4]{81}=\boxed{3}</math>.
 
We know that <math>81^{-2^{-2}}</math> is equivalent to <math>81^{\frac{1}{-2^2}}=81^{\frac{1}{4}}</math>, which is the same as <math>\sqrt[4]{81}=\boxed{3}</math>.

Revision as of 15:04, 23 June 2016

Problem

The expression $(81)^{-2^{-2}}$ has the same value as:

$\textbf{(A)}\ \frac {1}{81} \qquad \textbf{(B) }\ \frac {1}{3} \qquad \textbf{(C) }\ 3 \qquad \textbf{(D) }\ 81\qquad \textbf{(E) }\ 81^4$

Solution

We know that $81^{-2^{-2}}$ is equivalent to $81^{\frac{1}{-2^2}}=81^{\frac{1}{4}}$, which is the same as $\sqrt[4]{81}=\boxed{3}$.